When building a math test, the teacher creates two difficult problems for every five easy problems. If the teacher is creating a test with 56 questions on it, how many questions will be difficult?

Answers

Answer 1

Answer:

So the problems are in sets of seven

2 dif+5 easy=7 questions

56/7=8 sets of question

8x2=16

8x5=40

16 difficult questions

Answer 2
22 answers would be difficult

Related Questions

plz correct ans only thx for the help

Answers

Answer:

The width is 5m (A)

Step-by-step explanation:

The formula for area is length times width times height.

LxWxH

I hope this helps!

The area of a floor is 49m2. If the length is 8m, what is the width?
ANSWER: A.) 40 ÷ 8 = 5 m
Explanation:
A floor is a rectangle
The area of a rectangle is length • width
So to find the width, divide the area by the length:
40 ÷ 8= 5 m

A container holds 1 3/4 of litres of water. How many containers can be filled from 49 litres of water?
ANSWER: 28 containers
Explanation: the question is asking how many 1 3/4 liters of water-filled containers will be needed to have a total of 49 litres of water.

49 litres ÷ 1 3/4 litres = 28

Please Help me :)
.
.
.
.
.

Answers

the correct answer is x = 20
The answer is 20 because the line u see x is 160 so the one kitty corner is also 160 and a straight line is 180 degrees so 180-160=20 so the answer is 20 degrees

Answer this fast and correct and you will get a thanks, a brainliest and a 5-star review along with 10 points!

Answers

The answer is 25% if you subtract 55 by 30 you will get 25
Answer:  50

====================================================

Work Shown:

55% = 55/100 = 0.55

55% of 200 = 0.55*200 = 110 people prefer dogs

30% of 200 = 0.30*200 = 60 people prefer cats

Subtract the results: 110-60 = 50

Multiply the first number by its reciprocal

try to explain in the most easiest way as possible plz and thank you ​

Answers

Answer:

when we multiply a number by its reciprocal we get-

For example- 2/1 × 1/2 = 1 . Every number has a reciprocal except 0 (1/0 is undefined)

Answer box1 : 42
Answer box2 : 1

I always remember REciprocal = REverse the fraction. So the of 6/7 is going to be 7/6 and so on.

Now you can just think of it like regular multiplication. You can spilt it into 2 problems (top and bottom)

6 x 7 = 42
7 x 6 = 42

Any number over itself always equals 1, so that means anytime you multiply a fraction by its own reciprocal, your answer will always be 1

I hope this helps :)

Please Help me with Q3!!

Answers

Try your best work hard
try your best!! work super hard

Find the missing dimensions

Answers

Answer:

short base: 6 yards

long base: 8 yards

Step-by-step explanation:

Our understanding of your figure is shown below.

The question says the "shortest side" and the "width" have the same dimension. If the "width" is a reference to "height 6 yards", then it seems the "shortest side" is 6 yards. Since the slant sides are longer than the height, the "shortest side" is also the "short base."

The short base is 6 yards.

__

The long base overhangs the short base by 1 yard on either end, so it is a total of 2 yards longer than the short base. It it 6+2 = 8 yards long.

The long base is 8 yards.

Hope this is correct, if not then feel free to let me know. I'm sorry in advance if it is incorrect.

6 yards and 8 yards!!!

PLZ HELP MEEEE NEED ANSWERS ASAP

Answers

Step-by-step explanation:

Divide them up, and then after that, the parenthesis mean that you need to multiply them! So I hope that helps!

Please mark as brainlist

Answer:

Step-by-step explanation:

57) Order of operation: First what is inside the parenthesis - so division and then subtraction

For fraction division use KCF method

Keep the first fraction

Change division to multiplication

Flip the second fraction

[tex](\frac{5}{8}[/tex] ÷ [tex]\frac{1}{4})[/tex] - ([tex]\frac{4}{5}[/tex] ÷ [tex]\frac{1}{2})[/tex] = [tex](\frac{5}{8}*\frac{4}{1})-(\frac{4}{5}*\frac{2}{1})[/tex]

                      [tex]= (\frac{5}{2})-(\frac{8}{5})[/tex]

                      [tex]= \frac{5*5}{2*5} -\frac{8*2}{5*2}\\\\=\frac{25}{10}-\frac{16}{10}\\\\= \frac{9}{10}\\[/tex]

59)

[tex]\frac{3}{4} +\frac{1}{3} = \frac{3*3}{4*3}+\frac{1*4}{3*4}\\[/tex]

         [tex]=\frac{9}{12}+\frac{4}{12}\\\\= \frac{9+4}{12}\\\\= \frac{13}{12}[/tex]

[tex](\frac{3}{4}+\frac{1}{3})-\frac{9}{15}=\frac{13}{12}-\frac{3}{5}\\[/tex]

                    [tex]= \frac{13*5}{12*5}-\frac{3*12}{5*12}\\\\= \frac{65}{60}-\frac{48}{60}\\\\= \frac{65-48}{60}\\\\= \frac{17}{60}[/tex]

One staple weighs 31 milligrams. If a box of staples holds 250 staples, how many grams of staples does the box hold?
A: 6.75
B: 7.75
C: 67.5
D: 77.5
E: 7,750
If you get this right i will give you the brainliest!

Answers

Answer:

7.75

Step-by-step explanation:

1 mg = 0.001 g

31 mg * 0.001 = 0.31 grams per staple

0.31 g * 250 = 7.75 grams per 250 staples

Answer:

b) 7.75

Step-by-step explanation:

convert 31 milligrams to grams = .031 grams

250 x .031 = 7.75 grams

pls help i beg i will give brainlist and points

Answers

Answer:

7. 1/10 or 10%

9.5/10

11.5/10

13.9/10

Step-by-step explanation:

10%
50%
50%
90%

I did the math

can someone help please (highlighted in blue)

Answers

[tex]\implies {\blue {\boxed {\boxed {\purple {\sf {B)\:10}}}}}}[/tex]

[tex]\large\mathfrak{{\pmb{\underline{\red{Step-by-step\:explanation}}{\red{:}}}}}[/tex]

[tex]4 + 3 \: ( \: 10 - {2}^{3} \: )[/tex]

➺[tex] \: 4 + 3 \: ( \: 10 - 2 \times 2 \times 2 \: )[/tex]

➺[tex] \: 4 + 3 \: ( \: 10 - 8 \: )[/tex]

➺[tex] \: 4 + 3 \: ( \: 2 \: )[/tex]

➺[tex] \: 4 + 6[/tex]

➺[tex] \: 10[/tex]

Note:-

[tex]\sf\pink{PEMDAS\: rule.}[/tex]

P = Parentheses

E = Exponents

M = Multiplication

D = Division

A = Addition

S = Subtraction

[tex]\large\mathfrak{{\pmb{\underline{\orange{Mystique35 }}{\orange{❦}}}}}[/tex]

Answer:

10

Step-by-step explanation:

Use

Parenthesis

Exponents

Multiplication(Left - Right)

Division(Left - Right)

Addition(Left - Right)

Subtraction(Left - Right)

The system of equations has no solution:
True or False?
y = (2/3)x - 5

4x - 6y = 30

Answers

It’s false because, a system of linear equations has one solution when the graphs intersect at a point.
The answer for this question would be false

9 hundreds = _____ tens

Answers

Answer:

90

Step-by-step explanation:

900 in groups of 10 would need 90 groups to equal 900.

Answer:

90

Step-by-step explanation:

We also have 9 tens, which means we can say 9×10=90. And we have 24 ones, which means we can say 24×1=24.

According to his GPS, it will take Lanny 1 hour to travel 40 miles. Which expression shows how to determine the speed in miles per hour?
40 miles ÷ 1 hour
1 hour ÷ 40 miles
1 mile ÷ 40 hours
40 hours ÷ 1 mile

Answers

Answer:

40 ÷ 1

Step-by-step explanation:

Speed = Distance : Time

Speed = 40 ÷ 1

40miles divided by 1 hour.
Miles goes above hours in the equation

Expand and combine like terms.

(4b^2+3)(4b^2-3)

Answers

Answer:

16b^4-9

Step-by-step explanation:

Factoring gives us 16b^4+12b^2-12^2-9. Simplifying gives us the answer which is 16b^4-9

Answer:

[tex]16b^4-9[/tex]

Step-by-step explanation:

When multiplying binomials, we can use a method called FOIL (First-Outside-Inside-Last).

For [tex](t_1+t_2)(t_3+t_4)[/tex], let:

[tex]t_1\implies \text{First Term},\\t_2\implies \text{Second Term},\\t_3\implies \text{Third Term},\\t_4\implies \text{Fourth Term}[/tex]

In the FOIL method, the steps go:

[tex]\text{\textbf{F}irst: Multiply the first and third terms (first term of each binomial)},\\\text{\textbf{O}utside: Multiply the first and fourth terms},\\\text{\textbf{I}nside: Multiply the second and third terms},\\\text{\textbf{L}ast: Multiply the second and fourth terms (last term of each binomial) }[/tex]

Given [tex](4b^2+3)(4b^2-3)[/tex], we have:

[tex]\text{First: }4b^2\cdot 4b^2,\\\text{Outside: }4b^2\cdot -3,\\\text{Inside: }3\cdot 4b^2,\\\text{Last: }3\cdot -3,\\\rightarrow 16b^4-12b^2+12b^2-9[/tex]

Combine like terms:

[tex]\boxed{16b^4-9}[/tex]

Help me with the image i sent

Answers

just get the area for all of them and add legth time width

Do area which is base x height x width and add the length

Select the correct answer.
Which table represents the increasing linear function with the greatest unit rate?
A.
x y
2 16
5 10
B.
x y
2 14
6 12
C.
x y
2 -24
5 -15
D.
x y
2 -12
6 -16
E.
x y
2 -19
6 -17

Answers

Answer:

E

Step-by-step explanation:

Answer:

Option c is the correct answer if the answer is correct plz mark me as brainliest.

What is the volume of the composite figure shown? Use 3.14 for π.

Answers

Answer:

25.12

Step-by-step explanation:

the formula of a cone is pi r^2 * h/3

r = radius and h = height

the radius is 2 and the height is 3

and the question says use 3.14 instead of pi so

we have

3.14 * 2^2 * 6/3

2*2 or 2^2 = 4

3.14 * 4 * 6/3

6/3 = 2

3.14 * 4 * 2

4*2 = 8

and 8 * 3.14 is 25.12

The answer to this question is 25.12

help pleaseeee. this is due in an 10 minutes

Answers

Answer:

(you'll have to read the explanation)

Step-by-step explanation:

There are 8 lines from 1 to 2, so plotting 1 1/2 (or 1.5) is right in between. I'm not sure how to add the picture back into the explanation, but I will try my best to explain. You want to plot your point for 1 1/2 4 lines after the 1. For the 2 1/4, you want to plot your point on the second line from the 2. Let me know if it doesn't make sense!

Answer: See the attached image below

1 1/2 is located at the exact midpoint between 1 and 2.

2 1/4 is located 2 tickmarks to the right of 2.

===================================================

Explanation:

Think of 1/2 as 4/8

The number 1 1/2 is the same as 1 4/8

So we start at 1 and move 4 tickmarks to the right to get at the exact midpoint (note there are 8 tickmarked spaces between any two whole numbers).

This places us exactly at the midpoint between 1 and 2.

-----------------

To plot 2 1/4, we will effectively plot 2 2/8 since 1/4 = 2/8

We start at 2 and then move 2 tickmarks to the right to arrive at 2 2/8

If a Car can drive 35 miles on 7 gallons of gas, how many miles can it travel on 2 gallons of gas (assuming miles per gallon is proportional)? Give and explain your answer.

Answers

Answer:

10 miles

Step-by-step explanation:

35/7 is 5 so therefore if you do 5+5+5+5+5+5+5 is 35

so that means 2 gallons which is 10

10 miles! if you divide 35 by 7 you get 5 so that mean it drives 5 miles per gallon with 2 gallons it will drive 10 miles

A.1.5
B.9
C.20
D.22
plsss answer as fast as u can

the first person that answers and there answer is correct i will give u brainliest

Answers

Answer:

x=50 y=10

Step-by-step explanation:

the answer is D, because when you look at the chart






Rule 1: Multiply by 2, then add one third starting from 1. Rule 2: Add one half, then multiply by 4 starting from 0. What is the fourth ordered pair using the two sequences? (5, 10) (four and two thirds, 42) (21, 170) (two and one third, 2)

Answers

2 is the correct answer 2 is the correct answer
the answer your looking for is 2

(Geometry Question) : Given the points A(-3, -4) and B(2, 0), find the coordinates of the point P on directed line segment AB that partitions AB in the ratio 2:3.

Answers

Answer-7:4


Have a good day
7:4
Mark me Branalist pls

How do I put this into a paragraph , you get 76 points so please answer

Answers

Write it all together I think

Answer:

put it all together and write normally

Step-by-step explanation:

Other Questions
Each Sumerian city was the center of an independent what are the role of public participation perequisites in the development.write berifly. Trong giai on bin ng kinh t ti Vit Nam nm 2008 , trong hon cnh lm pht tang cao , c kin cho rng s lng ngn hng thng mi ti Vit Nam qu nhiu (39 ngn hng , tng ng s ngn hng ca Indonesia, nhiu hn Malaysia, Philipines ), to ra lng tin bt t ln , gp phn lm tng t l lm pht . Anh/ Ch hy nu nhn xt ca mnh v kin ny Within a year of the first vape devices being released, the World Health Organization released a warning that A 52 kg child on a swing is travelling at 6 m/s . What is his gravitational potential energy if he has 1000 J of the mechanical energy? A triangle has sides measuring 2 inches and 7 inches. If x represents thelength in inches of the third side, which inequality gives the range of possiblevalues for x?O A. 5sxs 9O B. 2 sxs7O C. 2OD. 5 If I eat 20 cupcakes and A=chicken nuggies what color is the imposter? True or false. extra points if you explain your answer. in a market economy decisions about which goods are produced are based on Given that abcd ~Jklm. Find the value of x, y, and z Using divisibility tests, check whether the number 240720 is divisible by2, 3, 4, 5, 6, 8, 9, 10 and 11. (Give reason) The absolute value of -7 x( 3x - 2y + 4z)x = -2, y = 4, and z = -3 In your own words, do you think its necessary for cultures to have a coming of age ceremony ? Why or why not? He felt( cold ) and ( still ) and ( hungry) , What literary device is used can someone tell me what the diffrence of 8 through 5 is helpppp please!!!!!! PLZZZZ I really need help! Which function rule represents the best line of fit for the data in the plot? Each month, Kelsey donates 1/5 of her allowance to her school for supplies. 1/2 of that amount goes to the chorus class. How much of her allowance goes to supplies for the chorus class?Please show your work.Thank you :) Help pls!!!!!!!!!!!!! trigonometry is it cos sin or tan Bell Corporation reports that at an activity level of 8,700 units, its total variable cost is $653,109, and its total fixed cost is $658,416. Required: For the activity level of 8,800 units, assume this level is within the relevant range. Compute: the total variable cost, the total fixed cost, the total cost, the average variable cost per unit, the average fixed cost per unit, and the average total cost per unit. There is not a word length requirement for this question; however, you must show your work.